Finding floor of reciprocal sumFind floor of sum $sum_k=1^80 k^-1/2$How should I handle the limit with the floor function?Is it possible to find sum of this series?Sum of a decreasing geometric series of integersIf $ A=frac12sqrt1+frac13sqrt2+frac14sqrt3+…+frac1100sqrt99;,$ Then $lfloor A rfloor =$floor sum of first $11$ terms of Harmonic SeriesWhat is the integer part of the sum of the reciprocals of the first $2017$ positive integers?On a series involving harmonic numbersSum of given telescopic seriesSum of series $sum^10_i=1ibigg(frac1^21+i+frac2^22+i+cdots cdots +frac10^210+ibigg)$Floor value of sum whose general term satisfy recursive relation.

Are modes in jazz primarily a melody thing?

Appropriate age to involve kids in life changing decisions

Why is the blank symbol not considered part of the input alphabet of a Turing machine?

Assuming a normal distribution: what is the sd for a given mean?

How do I minimise waste on a flight?

Can anyone identify this unknown 1988 PC card from The Palantir Corporation?

shebang or not shebang

Is it safe to keep the GPU on 100% utilization for a very long time?

Is there a reason why Turkey took the Balkan territories of the Ottoman Empire, instead of Greece or another of the Balkan states?

What is the Ancient One's mistake?

How to increase row height of a table and vertically "align middle"?

What is the meaning of "matter" in physics?

Convert Numbers To Emoji Math

Scaling rounded rectangles in Illustrator

Where do 5 or more U.S. counties meet in a single point?

The unknown and unexplained in science fiction

Do the Zhentarim fire members for killing fellow members?

Antivirus for Ubuntu 18.04

What chord could the notes 'F A♭ E♭' form?

How could a humanoid creature completely form within the span of 24 hours?

How to make a kid's bike easier to pedal

Gift for mentor after his thesis defense?

Extracting the parent, leaf, and extension from a valid path

Why were the rules for Proliferate changed?



Finding floor of reciprocal sum


Find floor of sum $sum_k=1^80 k^-1/2$How should I handle the limit with the floor function?Is it possible to find sum of this series?Sum of a decreasing geometric series of integersIf $ A=frac12sqrt1+frac13sqrt2+frac14sqrt3+…+frac1100sqrt99;,$ Then $lfloor A rfloor =$floor sum of first $11$ terms of Harmonic SeriesWhat is the integer part of the sum of the reciprocals of the first $2017$ positive integers?On a series involving harmonic numbersSum of given telescopic seriesSum of series $sum^10_i=1ibigg(frac1^21+i+frac2^22+i+cdots cdots +frac10^210+ibigg)$Floor value of sum whose general term satisfy recursive relation.













4












$begingroup$



Evaluation of



$$bigg lfloor frac1sqrt[3]1+frac1sqrt[3]2^2+frac1sqrt[3]3^2+cdots +frac1sqrt[3](1000)^2biggrfloor$$



Where $lfloor xrfloor $ is the floor of $x$




Try: It seems like we can solve it using Telescopic sums and that the sum lies between $2$ Telescopic sums, but could not figure out how to solve it.



Could someone help me to solve it? Thanks.










share|cite|improve this question











$endgroup$











  • $begingroup$
    Note the definition of the Generalized Harmonic numbers: $$H_N^(s)=sum_n=1^Nfrac1n^s$$ Your value is $$lfloor H_1000^(2/3) rfloor$$
    $endgroup$
    – clathratus
    Apr 28 at 18:25







  • 3




    $begingroup$
    I don’t think there is something telescopic here. Comparing this sum with an integral, and bounding the error term might help.
    $endgroup$
    – HAMIDINE SOUMARE
    Apr 28 at 18:26










  • $begingroup$
    In particular, you can check the answer of the user hypergeometric at math.stackexchange.com/questions/2570782/…. Follow his idea in your case. You can also work with Abel (partial) summation to estimate your sum.
    $endgroup$
    – Stelios Sachpazis
    Apr 28 at 18:28











  • $begingroup$
    The answer is $27$. See here
    $endgroup$
    – clathratus
    Apr 28 at 18:30















4












$begingroup$



Evaluation of



$$bigg lfloor frac1sqrt[3]1+frac1sqrt[3]2^2+frac1sqrt[3]3^2+cdots +frac1sqrt[3](1000)^2biggrfloor$$



Where $lfloor xrfloor $ is the floor of $x$




Try: It seems like we can solve it using Telescopic sums and that the sum lies between $2$ Telescopic sums, but could not figure out how to solve it.



Could someone help me to solve it? Thanks.










share|cite|improve this question











$endgroup$











  • $begingroup$
    Note the definition of the Generalized Harmonic numbers: $$H_N^(s)=sum_n=1^Nfrac1n^s$$ Your value is $$lfloor H_1000^(2/3) rfloor$$
    $endgroup$
    – clathratus
    Apr 28 at 18:25







  • 3




    $begingroup$
    I don’t think there is something telescopic here. Comparing this sum with an integral, and bounding the error term might help.
    $endgroup$
    – HAMIDINE SOUMARE
    Apr 28 at 18:26










  • $begingroup$
    In particular, you can check the answer of the user hypergeometric at math.stackexchange.com/questions/2570782/…. Follow his idea in your case. You can also work with Abel (partial) summation to estimate your sum.
    $endgroup$
    – Stelios Sachpazis
    Apr 28 at 18:28











  • $begingroup$
    The answer is $27$. See here
    $endgroup$
    – clathratus
    Apr 28 at 18:30













4












4








4


2



$begingroup$



Evaluation of



$$bigg lfloor frac1sqrt[3]1+frac1sqrt[3]2^2+frac1sqrt[3]3^2+cdots +frac1sqrt[3](1000)^2biggrfloor$$



Where $lfloor xrfloor $ is the floor of $x$




Try: It seems like we can solve it using Telescopic sums and that the sum lies between $2$ Telescopic sums, but could not figure out how to solve it.



Could someone help me to solve it? Thanks.










share|cite|improve this question











$endgroup$





Evaluation of



$$bigg lfloor frac1sqrt[3]1+frac1sqrt[3]2^2+frac1sqrt[3]3^2+cdots +frac1sqrt[3](1000)^2biggrfloor$$



Where $lfloor xrfloor $ is the floor of $x$




Try: It seems like we can solve it using Telescopic sums and that the sum lies between $2$ Telescopic sums, but could not figure out how to solve it.



Could someone help me to solve it? Thanks.







sequences-and-series






share|cite|improve this question















share|cite|improve this question













share|cite|improve this question




share|cite|improve this question








edited Apr 28 at 18:46









Stelios Sachpazis

1,085416




1,085416










asked Apr 28 at 18:22









DXTDXT

6,0202733




6,0202733











  • $begingroup$
    Note the definition of the Generalized Harmonic numbers: $$H_N^(s)=sum_n=1^Nfrac1n^s$$ Your value is $$lfloor H_1000^(2/3) rfloor$$
    $endgroup$
    – clathratus
    Apr 28 at 18:25







  • 3




    $begingroup$
    I don’t think there is something telescopic here. Comparing this sum with an integral, and bounding the error term might help.
    $endgroup$
    – HAMIDINE SOUMARE
    Apr 28 at 18:26










  • $begingroup$
    In particular, you can check the answer of the user hypergeometric at math.stackexchange.com/questions/2570782/…. Follow his idea in your case. You can also work with Abel (partial) summation to estimate your sum.
    $endgroup$
    – Stelios Sachpazis
    Apr 28 at 18:28











  • $begingroup$
    The answer is $27$. See here
    $endgroup$
    – clathratus
    Apr 28 at 18:30
















  • $begingroup$
    Note the definition of the Generalized Harmonic numbers: $$H_N^(s)=sum_n=1^Nfrac1n^s$$ Your value is $$lfloor H_1000^(2/3) rfloor$$
    $endgroup$
    – clathratus
    Apr 28 at 18:25







  • 3




    $begingroup$
    I don’t think there is something telescopic here. Comparing this sum with an integral, and bounding the error term might help.
    $endgroup$
    – HAMIDINE SOUMARE
    Apr 28 at 18:26










  • $begingroup$
    In particular, you can check the answer of the user hypergeometric at math.stackexchange.com/questions/2570782/…. Follow his idea in your case. You can also work with Abel (partial) summation to estimate your sum.
    $endgroup$
    – Stelios Sachpazis
    Apr 28 at 18:28











  • $begingroup$
    The answer is $27$. See here
    $endgroup$
    – clathratus
    Apr 28 at 18:30















$begingroup$
Note the definition of the Generalized Harmonic numbers: $$H_N^(s)=sum_n=1^Nfrac1n^s$$ Your value is $$lfloor H_1000^(2/3) rfloor$$
$endgroup$
– clathratus
Apr 28 at 18:25





$begingroup$
Note the definition of the Generalized Harmonic numbers: $$H_N^(s)=sum_n=1^Nfrac1n^s$$ Your value is $$lfloor H_1000^(2/3) rfloor$$
$endgroup$
– clathratus
Apr 28 at 18:25





3




3




$begingroup$
I don’t think there is something telescopic here. Comparing this sum with an integral, and bounding the error term might help.
$endgroup$
– HAMIDINE SOUMARE
Apr 28 at 18:26




$begingroup$
I don’t think there is something telescopic here. Comparing this sum with an integral, and bounding the error term might help.
$endgroup$
– HAMIDINE SOUMARE
Apr 28 at 18:26












$begingroup$
In particular, you can check the answer of the user hypergeometric at math.stackexchange.com/questions/2570782/…. Follow his idea in your case. You can also work with Abel (partial) summation to estimate your sum.
$endgroup$
– Stelios Sachpazis
Apr 28 at 18:28





$begingroup$
In particular, you can check the answer of the user hypergeometric at math.stackexchange.com/questions/2570782/…. Follow his idea in your case. You can also work with Abel (partial) summation to estimate your sum.
$endgroup$
– Stelios Sachpazis
Apr 28 at 18:28













$begingroup$
The answer is $27$. See here
$endgroup$
– clathratus
Apr 28 at 18:30




$begingroup$
The answer is $27$. See here
$endgroup$
– clathratus
Apr 28 at 18:30










3 Answers
3






active

oldest

votes


















1












$begingroup$

Note that $displaystyle sum_k=1^1000frac3sqrt[3](k+1)^2+sqrt[3]k(k+1)+sqrt[3]k^2<sum_k=1^1000frac1sqrt[3]k^2<1+sum_k=2^1000frac3sqrt[3](k-1)^2+sqrt[3]k(k-1)+sqrt[3]k^2$.



$displaystyle sum_k=1^1000frac3sqrt[3](k+1)^2+sqrt[3]k(k+1)+sqrt[3]k^2=sum_k=1^1000frac3(sqrt[3]k+1-sqrt[3]k)(k+1)-k=3(sqrt[3]1001-1)>27$



$displaystyle 1+sum_k=2^1000frac3sqrt[3](k-1)^2+sqrt[3]k(k-1)+sqrt[3]k^2=1+sum_k=2^1000frac3(sqrt[3]k-sqrt[3]k-1)(k-1)-k=1+3(sqrt[3]1000-1)=28$



So, $displaystyle leftlfloor sum_k=1^1000frac1sqrt[3]k^2rightrfloor=27$.






share|cite|improve this answer











$endgroup$




















    4












    $begingroup$

    One approach is to approximate the sum with an integral, and show that the error is bounded by a number less than 1.



    In particular, let $$H_1000^(2/3) = sum_k=1^1000 k^-2/3, quad I = int_x=1^1000 x^-2/3 , dx.$$ Then we know $$I le H_1000^(2/3) < I+1.$$ But $I = 27$, and we are done.






    share|cite|improve this answer









    $endgroup$




















      4












      $begingroup$

      You can bound this summation by two integrals;
      $$int_1^1001 x^-2/3mathrmdxltsum_k=1^1000k^-2/3lt1+int_1^1000x^-2/3mathrmdx$$
      Hence we have
      $$27lt3sqrt[3]1001-3ltsum_k=1^1000k^-2/3lt28$$
      So as the value of the sum is strictly between $27$ and $28$, the floor of the sum is $27$.






      share|cite|improve this answer









      $endgroup$













        Your Answer








        StackExchange.ready(function()
        var channelOptions =
        tags: "".split(" "),
        id: "69"
        ;
        initTagRenderer("".split(" "), "".split(" "), channelOptions);

        StackExchange.using("externalEditor", function()
        // Have to fire editor after snippets, if snippets enabled
        if (StackExchange.settings.snippets.snippetsEnabled)
        StackExchange.using("snippets", function()
        createEditor();
        );

        else
        createEditor();

        );

        function createEditor()
        StackExchange.prepareEditor(
        heartbeatType: 'answer',
        autoActivateHeartbeat: false,
        convertImagesToLinks: true,
        noModals: true,
        showLowRepImageUploadWarning: true,
        reputationToPostImages: 10,
        bindNavPrevention: true,
        postfix: "",
        imageUploader:
        brandingHtml: "Powered by u003ca class="icon-imgur-white" href="https://imgur.com/"u003eu003c/au003e",
        contentPolicyHtml: "User contributions licensed under u003ca href="https://creativecommons.org/licenses/by-sa/3.0/"u003ecc by-sa 3.0 with attribution requiredu003c/au003e u003ca href="https://stackoverflow.com/legal/content-policy"u003e(content policy)u003c/au003e",
        allowUrls: true
        ,
        noCode: true, onDemand: true,
        discardSelector: ".discard-answer"
        ,immediatelyShowMarkdownHelp:true
        );



        );













        draft saved

        draft discarded


















        StackExchange.ready(
        function ()
        StackExchange.openid.initPostLogin('.new-post-login', 'https%3a%2f%2fmath.stackexchange.com%2fquestions%2f3205926%2ffinding-floor-of-reciprocal-sum%23new-answer', 'question_page');

        );

        Post as a guest















        Required, but never shown

























        3 Answers
        3






        active

        oldest

        votes








        3 Answers
        3






        active

        oldest

        votes









        active

        oldest

        votes






        active

        oldest

        votes









        1












        $begingroup$

        Note that $displaystyle sum_k=1^1000frac3sqrt[3](k+1)^2+sqrt[3]k(k+1)+sqrt[3]k^2<sum_k=1^1000frac1sqrt[3]k^2<1+sum_k=2^1000frac3sqrt[3](k-1)^2+sqrt[3]k(k-1)+sqrt[3]k^2$.



        $displaystyle sum_k=1^1000frac3sqrt[3](k+1)^2+sqrt[3]k(k+1)+sqrt[3]k^2=sum_k=1^1000frac3(sqrt[3]k+1-sqrt[3]k)(k+1)-k=3(sqrt[3]1001-1)>27$



        $displaystyle 1+sum_k=2^1000frac3sqrt[3](k-1)^2+sqrt[3]k(k-1)+sqrt[3]k^2=1+sum_k=2^1000frac3(sqrt[3]k-sqrt[3]k-1)(k-1)-k=1+3(sqrt[3]1000-1)=28$



        So, $displaystyle leftlfloor sum_k=1^1000frac1sqrt[3]k^2rightrfloor=27$.






        share|cite|improve this answer











        $endgroup$

















          1












          $begingroup$

          Note that $displaystyle sum_k=1^1000frac3sqrt[3](k+1)^2+sqrt[3]k(k+1)+sqrt[3]k^2<sum_k=1^1000frac1sqrt[3]k^2<1+sum_k=2^1000frac3sqrt[3](k-1)^2+sqrt[3]k(k-1)+sqrt[3]k^2$.



          $displaystyle sum_k=1^1000frac3sqrt[3](k+1)^2+sqrt[3]k(k+1)+sqrt[3]k^2=sum_k=1^1000frac3(sqrt[3]k+1-sqrt[3]k)(k+1)-k=3(sqrt[3]1001-1)>27$



          $displaystyle 1+sum_k=2^1000frac3sqrt[3](k-1)^2+sqrt[3]k(k-1)+sqrt[3]k^2=1+sum_k=2^1000frac3(sqrt[3]k-sqrt[3]k-1)(k-1)-k=1+3(sqrt[3]1000-1)=28$



          So, $displaystyle leftlfloor sum_k=1^1000frac1sqrt[3]k^2rightrfloor=27$.






          share|cite|improve this answer











          $endgroup$















            1












            1








            1





            $begingroup$

            Note that $displaystyle sum_k=1^1000frac3sqrt[3](k+1)^2+sqrt[3]k(k+1)+sqrt[3]k^2<sum_k=1^1000frac1sqrt[3]k^2<1+sum_k=2^1000frac3sqrt[3](k-1)^2+sqrt[3]k(k-1)+sqrt[3]k^2$.



            $displaystyle sum_k=1^1000frac3sqrt[3](k+1)^2+sqrt[3]k(k+1)+sqrt[3]k^2=sum_k=1^1000frac3(sqrt[3]k+1-sqrt[3]k)(k+1)-k=3(sqrt[3]1001-1)>27$



            $displaystyle 1+sum_k=2^1000frac3sqrt[3](k-1)^2+sqrt[3]k(k-1)+sqrt[3]k^2=1+sum_k=2^1000frac3(sqrt[3]k-sqrt[3]k-1)(k-1)-k=1+3(sqrt[3]1000-1)=28$



            So, $displaystyle leftlfloor sum_k=1^1000frac1sqrt[3]k^2rightrfloor=27$.






            share|cite|improve this answer











            $endgroup$



            Note that $displaystyle sum_k=1^1000frac3sqrt[3](k+1)^2+sqrt[3]k(k+1)+sqrt[3]k^2<sum_k=1^1000frac1sqrt[3]k^2<1+sum_k=2^1000frac3sqrt[3](k-1)^2+sqrt[3]k(k-1)+sqrt[3]k^2$.



            $displaystyle sum_k=1^1000frac3sqrt[3](k+1)^2+sqrt[3]k(k+1)+sqrt[3]k^2=sum_k=1^1000frac3(sqrt[3]k+1-sqrt[3]k)(k+1)-k=3(sqrt[3]1001-1)>27$



            $displaystyle 1+sum_k=2^1000frac3sqrt[3](k-1)^2+sqrt[3]k(k-1)+sqrt[3]k^2=1+sum_k=2^1000frac3(sqrt[3]k-sqrt[3]k-1)(k-1)-k=1+3(sqrt[3]1000-1)=28$



            So, $displaystyle leftlfloor sum_k=1^1000frac1sqrt[3]k^2rightrfloor=27$.







            share|cite|improve this answer














            share|cite|improve this answer



            share|cite|improve this answer








            edited Apr 29 at 3:47

























            answered Apr 29 at 3:36









            CY AriesCY Aries

            17.8k11743




            17.8k11743





















                4












                $begingroup$

                One approach is to approximate the sum with an integral, and show that the error is bounded by a number less than 1.



                In particular, let $$H_1000^(2/3) = sum_k=1^1000 k^-2/3, quad I = int_x=1^1000 x^-2/3 , dx.$$ Then we know $$I le H_1000^(2/3) < I+1.$$ But $I = 27$, and we are done.






                share|cite|improve this answer









                $endgroup$

















                  4












                  $begingroup$

                  One approach is to approximate the sum with an integral, and show that the error is bounded by a number less than 1.



                  In particular, let $$H_1000^(2/3) = sum_k=1^1000 k^-2/3, quad I = int_x=1^1000 x^-2/3 , dx.$$ Then we know $$I le H_1000^(2/3) < I+1.$$ But $I = 27$, and we are done.






                  share|cite|improve this answer









                  $endgroup$















                    4












                    4








                    4





                    $begingroup$

                    One approach is to approximate the sum with an integral, and show that the error is bounded by a number less than 1.



                    In particular, let $$H_1000^(2/3) = sum_k=1^1000 k^-2/3, quad I = int_x=1^1000 x^-2/3 , dx.$$ Then we know $$I le H_1000^(2/3) < I+1.$$ But $I = 27$, and we are done.






                    share|cite|improve this answer









                    $endgroup$



                    One approach is to approximate the sum with an integral, and show that the error is bounded by a number less than 1.



                    In particular, let $$H_1000^(2/3) = sum_k=1^1000 k^-2/3, quad I = int_x=1^1000 x^-2/3 , dx.$$ Then we know $$I le H_1000^(2/3) < I+1.$$ But $I = 27$, and we are done.







                    share|cite|improve this answer












                    share|cite|improve this answer



                    share|cite|improve this answer










                    answered Apr 28 at 18:40









                    heropupheropup

                    66.2k866104




                    66.2k866104





















                        4












                        $begingroup$

                        You can bound this summation by two integrals;
                        $$int_1^1001 x^-2/3mathrmdxltsum_k=1^1000k^-2/3lt1+int_1^1000x^-2/3mathrmdx$$
                        Hence we have
                        $$27lt3sqrt[3]1001-3ltsum_k=1^1000k^-2/3lt28$$
                        So as the value of the sum is strictly between $27$ and $28$, the floor of the sum is $27$.






                        share|cite|improve this answer









                        $endgroup$

















                          4












                          $begingroup$

                          You can bound this summation by two integrals;
                          $$int_1^1001 x^-2/3mathrmdxltsum_k=1^1000k^-2/3lt1+int_1^1000x^-2/3mathrmdx$$
                          Hence we have
                          $$27lt3sqrt[3]1001-3ltsum_k=1^1000k^-2/3lt28$$
                          So as the value of the sum is strictly between $27$ and $28$, the floor of the sum is $27$.






                          share|cite|improve this answer









                          $endgroup$















                            4












                            4








                            4





                            $begingroup$

                            You can bound this summation by two integrals;
                            $$int_1^1001 x^-2/3mathrmdxltsum_k=1^1000k^-2/3lt1+int_1^1000x^-2/3mathrmdx$$
                            Hence we have
                            $$27lt3sqrt[3]1001-3ltsum_k=1^1000k^-2/3lt28$$
                            So as the value of the sum is strictly between $27$ and $28$, the floor of the sum is $27$.






                            share|cite|improve this answer









                            $endgroup$



                            You can bound this summation by two integrals;
                            $$int_1^1001 x^-2/3mathrmdxltsum_k=1^1000k^-2/3lt1+int_1^1000x^-2/3mathrmdx$$
                            Hence we have
                            $$27lt3sqrt[3]1001-3ltsum_k=1^1000k^-2/3lt28$$
                            So as the value of the sum is strictly between $27$ and $28$, the floor of the sum is $27$.







                            share|cite|improve this answer












                            share|cite|improve this answer



                            share|cite|improve this answer










                            answered Apr 28 at 18:41









                            Peter ForemanPeter Foreman

                            9,5241321




                            9,5241321



























                                draft saved

                                draft discarded
















































                                Thanks for contributing an answer to Mathematics Stack Exchange!


                                • Please be sure to answer the question. Provide details and share your research!

                                But avoid


                                • Asking for help, clarification, or responding to other answers.

                                • Making statements based on opinion; back them up with references or personal experience.

                                Use MathJax to format equations. MathJax reference.


                                To learn more, see our tips on writing great answers.




                                draft saved


                                draft discarded














                                StackExchange.ready(
                                function ()
                                StackExchange.openid.initPostLogin('.new-post-login', 'https%3a%2f%2fmath.stackexchange.com%2fquestions%2f3205926%2ffinding-floor-of-reciprocal-sum%23new-answer', 'question_page');

                                );

                                Post as a guest















                                Required, but never shown





















































                                Required, but never shown














                                Required, but never shown












                                Required, but never shown







                                Required, but never shown

































                                Required, but never shown














                                Required, but never shown












                                Required, but never shown







                                Required, but never shown







                                Popular posts from this blog

                                Wikipedia:Vital articles Мазмуну Biography - Өмүр баян Philosophy and psychology - Философия жана психология Religion - Дин Social sciences - Коомдук илимдер Language and literature - Тил жана адабият Science - Илим Technology - Технология Arts and recreation - Искусство жана эс алуу History and geography - Тарых жана география Навигация менюсу

                                Bruxelas-Capital Índice Historia | Composición | Situación lingüística | Clima | Cidades irmandadas | Notas | Véxase tamén | Menú de navegacióneO uso das linguas en Bruxelas e a situación do neerlandés"Rexión de Bruxelas Capital"o orixinalSitio da rexiónPáxina de Bruselas no sitio da Oficina de Promoción Turística de Valonia e BruxelasMapa Interactivo da Rexión de Bruxelas-CapitaleeWorldCat332144929079854441105155190212ID28008674080552-90000 0001 0666 3698n94104302ID540940339365017018237

                                What should I write in an apology letter, since I have decided not to join a company after accepting an offer letterShould I keep looking after accepting a job offer?What should I do when I've been verbally told I would get an offer letter, but still haven't gotten one after 4 weeks?Do I accept an offer from a company that I am not likely to join?New job hasn't confirmed starting date and I want to give current employer as much notice as possibleHow should I address my manager in my resignation letter?HR delayed background verification, now jobless as resignedNo email communication after accepting a formal written offer. How should I phrase the call?What should I do if after receiving a verbal offer letter I am informed that my written job offer is put on hold due to some internal issues?Should I inform the current employer that I am about to resign within 1-2 weeks since I have signed the offer letter and waiting for visa?What company will do, if I send their offer letter to another company